diff --git a/BDA 3.10.11.ipynb b/BDA 3.10.11.ipynb index fcb0578..4c37fc8 100644 --- a/BDA 3.10.11.ipynb +++ b/BDA 3.10.11.ipynb @@ -2,10 +2,7 @@ "cells": [ { "cell_type": "markdown", - "metadata": { - "deletable": true, - "editable": true - }, + "metadata": {}, "source": [ "Problem 3.10.5. 5. Rounded data: it is a common problem for measurements to be observed in rounded form (for a review, see Heitjan, 1989). For a simple example, suppose we weigh an object five times and measure weights, rounded to the nearest pound, of 10, 10, 12, 11, 9. Assume the unrounded measurements are normally distributed with a noninformative prior distribution on the mean ยต and variance $\\sigma^2$. \n", "\n", @@ -21,29 +18,21 @@ }, { "cell_type": "code", - "execution_count": 32, - "metadata": { - "collapsed": true, - "deletable": true, - "editable": true - }, + "execution_count": 2, + "metadata": {}, "outputs": [], "source": [ "import numpy as np\n", - "from scipy.stats import norm,chi2,invgamma,t\n", + "from scipy.stats import norm,chi2,invgamma,t,uniform\n", "import matplotlib.pyplot as plt\n", "from math import log,sqrt\n", - "%matplotlib inline\n" + "\n" ] }, { "cell_type": "code", - "execution_count": 33, - "metadata": { - "collapsed": false, - "deletable": true, - "editable": true - }, + "execution_count": 3, + "metadata": {}, "outputs": [ { "name": "stdout", @@ -63,10 +52,7 @@ }, { "cell_type": "markdown", - "metadata": { - "deletable": true, - "editable": true - }, + "metadata": {}, "source": [ "The inverse chi-square distribution which is the conjugate prior for the variance is equivalent to the inverse gamma distribution with the correct parameters: in the notation of scipy, it appears that\n", "Inv-chi^2(df-1,s^2)=invgamma((df-1)/2,((df-1)/2)s^2). Note the weird thing that the variance of this doesn't exist in the case where df=5." @@ -74,26 +60,32 @@ }, { "cell_type": "code", - "execution_count": 39, - "metadata": { - "collapsed": false, - "deletable": true, - "editable": true - }, + "execution_count": 4, + "metadata": {}, "outputs": [ { "name": "stdout", "output_type": "stream", "text": [ - "2.54758506289\n", - "0.206923011397 362.907443474\n" + "2.599274021823669\n", + "[0.96947339 1.55234798 2.69693603]\n" ] }, { "data": { - "image/png": "iVBORw0KGgoAAAANSUhEUgAAAXoAAAD8CAYAAAB5Pm/hAAAABHNCSVQICAgIfAhkiAAAAAlwSFlz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\nnEEvSY0z6CWpcQa9JDXOoJekxv0/Mdhzd8ElwZUAAAAASUVORK5CYII=\n", "text/plain": [ - "" + "[]" + ] + }, + "execution_count": 4, + "metadata": {}, + "output_type": "execute_result" + }, + { + "data": { + "image/png": "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\n", + "text/plain": [ + "
" ] }, "metadata": {}, @@ -105,17 +97,23 @@ "samps=sample_v.rvs(10000)\n", "# as a check, the mean is supposed to be 2s^2\n", "print(samps.mean())\n", - "print(samps.min(),samps.max())\n", - "junk=plt.hist([x for x in samps if x<25],bins=1000)\n", - " " + "print(np.percentile(samps,[25,50,75]))\n", + "plt.figure(1)\n", + "plt.xlim(0,10)\n", + "junk=plt.hist(samps,bins=1000,density=True)\n", + "##### Gelman proposes actually sampling from chisquare and taking reciprocals -- gives the same\n", + "##### result, so we got the normalization right.\n", + "gelman=chi2(4)\n", + "samps_g=4*v/gelman.rvs(10000)\n", + "samps_g.mean()\n", + "np.percentile(samps_g,[25,50,75])\n", + "junk2=plt.hist(samps_g,bins=1000,density=True)\n", + "plt.plot(np.linspace(0,10,1000),sample_v.pdf(np.linspace(0,10,1000)))\n" ] }, { "cell_type": "markdown", - "metadata": { - "deletable": true, - "editable": true - }, + "metadata": {}, "source": [ "For the mean: for each variance in the distribution, we get a potential mean; the distribution in question is N(xbar,sigma^2/5) so we can choose from N(0,1) and adjust by z=(sqrt(5)(x-xbar)/sigma) so x=sigma/sqrt(5)z+xbar.\n", "\n", @@ -124,28 +122,21 @@ }, { "cell_type": "markdown", - "metadata": { - "deletable": true, - "editable": true - }, + "metadata": {}, "source": [ "##### " ] }, { "cell_type": "code", - "execution_count": 37, - "metadata": { - "collapsed": false, - "deletable": true, - "editable": true - }, + "execution_count": 5, + "metadata": {}, "outputs": [ { "data": { - "image/png": "iVBORw0KGgoAAAANSUhEUgAAAXoAAAD8CAYAAAB5Pm/hAAAABHNCSVQICAgIfAhkiAAAAAlwSFlz\nAAALEgAACxIB0t1+/AAAIABJREFUeJzt3XuYVNWZ7/HvayMoeEGlAwZUkKAICqgd1IyiOWYmOEYJ\njsmBXLyNIWiYmEwmiic5mZnHk0xMJuZygmHQGPUkLWrihWRA0JxRjKLSICCISIMioCJyNdwb3vlj\n7Z2uLrrp6qaq9q7dv8/z1LN2Ve2ueqG7f71r7bXXMndHRESy65CkCxARkdJS0IuIZJyCXkQk4xT0\nIiIZp6AXEck4Bb2ISMYp6EVEMk5BLyKScQp6EZGM65TUG/fo0cP79u2b1NuLiFSkefPmve/u1W35\nmsSCvm/fvtTV1SX19iIiFcnMVrX1a9R1IyKScQp6EZGMU9CLiGScgl5EJOMU9CIiGaegFxHJOAW9\niEjGKehFRDJOQS8iknEKepFYrSVdgUhJKOhFRDJOQS8iknEKehGRjFPQi4hkXEFBb2YjzWyZmdWb\n2cRmnu9hZk+Y2UIzW2Jm1xa/VBERaY9Wg97MqoBJwCXAIGCsmQ3K220CsNDdhwIXAT8ys85FrlVE\nRNqhkCP64UC9u690993AVGBU3j7vAkeamQFHABuBhqJWKlJKGlopGVZI0PcGVufcXxM9lusuwtH+\n28ArwE3uvi//hcxsnJnVmVnd+vXr21myiIi0RbFOxt4KLAI+DAwDfm5mR+Xv5O5T3L3G3Wuqq9u0\n5KGIiLRTIUG/Fjgh536f6LFcfwU87EE98AYwsDgliojIwSgk6OcCA8ysX3SCdQwwLW+f14CLAcys\nJ3AqsLKYhYqUhfrqJYM6tbaDuzeY2QRgJlAF3OPuS8xsfPT8ZOB7wK/MbBHhj8ct7v5+CesWEZEC\ntRr0AO4+HZie99jknO31wKeKW5qIiBSDrowVaY66cCRDFPQiIhmnoBcRyTgFvYhIxinoRUQyTkEv\nIpJxCnoRkYxT0IuIZJyCXkQk4wq6MlYks+bPh4ceghWEJXOOSbgekRJQ0EvHddddMH487IuWTpgO\n7LdQpkjlU9eNdEzPP98Y8tdfD6cD24E7AC2KIxmjoJeOp6EBbrghhPw3vxmO7G8mrKCwGfjOdxIu\nUKS4FPTS8TzwACxaBNXAv/5reKwKuA4wQvC/m1x5IsWmoJeOxR3uuCNsjwYe7dr4XG/gAmDv3rD6\ngkhGKOilY/nTn2DBAujZEz7WzPMjo/ZZYNu2MhYmUjoFBb2ZjTSzZWZWb2b7jUsws2+a2YLottjM\n9prZscUvV+Qg3X9/aK+7Dg5t5vmTgPPOgx3Aww+XsTCR0mk16M2sCpgEXAIMAsaa2aDcfdz9h+4+\nzN2HAbcCz7j7xlIULNJuO3c2hvcXvtDyftdeG9oHHyx9TSJlUMgR/XCg3t1XuvtuYCow6gD7jwUe\nKEZxIkU1axZs2QLDhsGCwS3vN3p0+M146inYqOMVqXyFBH1vYHXO/TXRY/sxs66EXs7ftfD8ODOr\nM7O69RqrLOX2hz+E9oorDrxfjx7hs2tDA0ybVvKyREqt2CdjLwOea6nbxt2nuHuNu9dUV1cX+a1F\nDsAdpkfr2196aev710TtjBklK0mkXAoJ+rXACTn3+0SPNWcM6raRNFq4ENauheOPhzPPbH3/IVH7\n5JNhuKVIBSsk6OcCA8ysn5l1JoT5fp9nzexo4ELg8eKWKFIE34vC/dJLwaz1/XsC/fvDpk0wd25J\nSxMptVaD3t0bgAmES0iWAg+5+xIzG29m43N2HQ3McncNPpb0WRK1I0cecLcmPvnJ0M7U1VNS2Qrq\no3f36e5+irv3d/fvRo9NdvfJOfvc6+5jSlWoSLvt2gXLo+0LL4TaAo7ooTHoZ80qSVki5aIrYyX7\n5s6FPcDgwWFETaHWjQrdPC8+Dzt2lKw8kVJT0Ev2zZ4d2gsvbNvXdQOGDIG9qJ9eKpqCXrLvmWdC\nO2JE4d02sfPPD+2f/lTcmkTKSEEv2bZnDzz3XNgeMaKwr8n9YxAH/bPP7v+cSIVQ0Eu2vfxymIWy\nF2EMfVvFQf/88xpPLxVLQS/Zk3vUPWdOaE9t52v16RMWKNm6FRYvPtjKRBKhoJdse/hroe1/EK9x\nStSqn14qlIJesm1F1J7czq+vNRgQbWvkjVQoBb1k18aNYe3XQ2k6W1NbxX8kFPRSoRT0kl11daHt\nC3Q6iNc5gbB4+NKlsPOgqxIpOwW9ZFd8BB73z7c0NLK1IZOdCWHvDm8UqTaRMlLQS3Y98u3Qtrd/\nPlf8Ggp6qUAKeskm98YTsQcz4ibWL2pXFuG1RMpMQS/ZtGYNbCHMV9OzCK8XH9Er6KUCKeglm+IT\nsScDxZi1oA9w2GGwjrAYiUgFKSjozWykmS0zs3ozm9jCPheZ2QIzW2JmzxS3TJE2WrQotH2L9Hqd\ngGHDwva8eUV6UZHyaDXozawKmARcAgwCxprZoLx9ugN3Ape7+2DgMyWoVaRwcdAfzPj5fEe9ENr4\n04JIhSjkiH44UO/uK919NzAVGJW3z+eAR9z9LQB3f6+4ZYq0URz0JxbxNU+K2oULi/iiIqVXSND3\nBlbn3F8TPZbrFOAYM3vazOaZ2VXFKlCkzXYCK1aEi5zaMWFli+KgX7CgiC8qUnoHc71g/uucDVwM\nHA7MMbMX3P313J3MbBwwDuDEE4t5qCUSqbVwKOIeDkeK9RMO4fUOAV5/PSwtePjhRXxxkdIp5Ih+\nLU17OvtEj+VaA8x0923u/j4wGxia/0LuPsXda9y9prq6ur01ixzYW1Fb7GOJzoRPCPv2acpiqSiF\nBP1cYICZ9TOzzsAYYFrePo8D55tZJzPrCpwDLC1uqSIFijsaS/GhUf30UoFa/WDr7g1mNgGYSej1\nvMfdl5jZ+Oj5ye6+1MyeABYB+4C73V2HPJKM+Ii+mCNuYicCz6N+eqkoBfVguvt0YHreY5Pz7v8Q\n+GHxShNpB6d0XTegI3qpSLoyVrJlI7AdOAo4ugSvH//xWLQonPAVqQAKesmW3G6bYkx9kK870LNn\nWEP2zTdL8AYixaegl2wpZbdNbGg0oEz99FIhFPSSLfGIm1KciI3FQa9+eqkQCnrJllVRW44jegW9\nVAgFvWTHzp3wDqFvPn+SjmJa+4XQKuilQijoJTtefTUMrzyecBVrqRwPdOkCb7wBW7aU8I1EikNB\nL9lRihkrm1MFDB4ctu/oXuI3Ezl4CnrJjnIFPcCQIaF968C7iaSBgl6yoxSLjbREQS8VpJiTuIok\n5zcGC3uE7XIe0a8+8G4iaaAjesmGLcD770NX4LgyvN/yT4R2NWHaYpEUU9BLNpR66oN8RwG9eoXV\nrFatam1vkUQp6CUbSjkHfUvi7pv43IBISinopfLVWnnmuMmnoJcKoaCXbCjlYiMtUdBLhSgo6M1s\npJktM7N6M5vYzPMXmdkWM1sQ3b5T/FJFWtBA4yrGCnqR/bQ6vNLMqoBJwF8TFgGfa2bT3P3VvF2f\ndfdPlaBGkQN7B9gL9AQOK+P7DhwYrpJdvhy2b4euXcv45iKFK+SIfjhQ7+4r3X03MBUYVdqyRNog\niW4bCPPdHE9YaepH3cr85iKFKyToe9P0spA1ND834MfMbJGZzTCzwUWpTqQQSYy4icXvqQunJMWK\ndTJ2PnCiuw8B/i/wWHM7mdk4M6szs7r169cX6a2lw0tixE0sfk9NhSApVkjQr6Xph+I+NJ76AsDd\nt7r7n6Pt6cChZtYj/4XcfYq717h7TXV19UGULZIjqa6b3PdU0EuKFRL0c4EBZtbPzDoDY4BpuTuY\nWS8zs2h7ePS6G4pdrMh+NmyATUAX4EMJvH8c9KsJffUiKdTqqBt3bzCzCcBMwhiDe9x9iZmNj56f\nDFwJ3GBmDcAOYIy7fuqlDF55JbR9SOaqkGOBY46BTZvg7behdymXthJpn4Jmr4y6Y6bnPTY5Z/vn\nwM+LW5pIAco5B31zjDCe/plnQi0KekkhXRkrlS3JoK+NZk/ThVOScgp6qWzlXGykJQp6STkFvVSu\nvXth8eKwnVTXDSjoJfUU9FK5VqyAHTvCQiNJXpg6eHDoq3/tNdi9O8FCRJqnoJfKlYZuG4Bu3cI8\nOw0NIexFUkZBL5Ur6RE3ueI/Nuq+kRRS0EvliUe7pCXoa62xBgW9pJCCXipXWrpuQEEvqaagl8q0\ndSu88Ua45K9X0sWgoJdUU9BLZYoDtQ8FXt9dYj2AI46Ad94BzcwqKaOgl8q0cGFok+6fjx0CHP/n\nsB3PvyOSEgp6qUxpC3rQyBtJLQW9VKY0Br366SWlFPRSefbR2D1yUqKVNKWgl5RS0EvleZcw9UGf\nPnBE0sXkiLtuliwJV8mKpISCXipPvGzf0KGJlrGfroTRNzt3Qn190tWI/EVBQW9mI81smZnVm9nE\nA+z3UTNrMLMri1eiSJ446IcNS7SMZumErKRQq0FvZlXAJOASYBAw1swGtbDf7cCsYhcp0sSqqE3b\nET2on15SqZAj+uFAvbuvdPfdwFRgVDP7/QPwO+C9ItYnsr+0dt2AjugllQoJ+t6ENe5ja6LH/sLM\negOjgV8UrzSRZmzYABuBrl2hf/+kq9mfjuglhYp1MvYnwC3uvu9AO5nZODOrM7O69bpMXNojHj9/\nxhlQVZVsLc3pBXTpAqtWwZYtSVcjAhQW9GtpOj9gn+ixXDXAVDN7E7gSuNPMPp3/Qu4+xd1r3L2m\nurq6nSVLhxYHfRq7bQCqCCtOQeMyhyIJKyTo5wIDzKyfmXUGxgDTcndw937u3tfd+wK/BW5098eK\nXq1I2oMetIaspE6rQe/uDcAEYCawFHjI3ZeY2XgzG1/qAkWaiIN+3VeSreNA4qD/3Y3J1iESKWiC\nV3efDkzPe2xyC/tec/BliTRjzx549dWwnaY5bvLFQf/WgXcTKRddGSuVY8kS2L07LMR9eNLFHEAc\n9KuBfQccnyBSFgp6qRzz5oW2X7JltKq6Gnr1gp3Az1I4Mkg6HAW9VI5KCfpaa5ye4Y1kSxEBBb1U\nkjjo+yZaRWFqakKroJcUUNBLZdizp3HETXxEX2uJldOqbf8ntCuTLUMEFPRSKZYuhV274ENAt6SL\nKcDJUfsG4J5kJSIKeqkQldI/HzsG6A5sB1bqsF6SpaCXylBpQW801lpXl2QlIgp6qQC1VlknYmMK\nekkJBb2k314aT8T2TbKQNor76RX0kjAFvaTf24TFwKuBI5Mupg3iI/p583SFrCRKQS/pF5/L7Jtk\nEe3QnXBS9oMP4A5dISvJUdBL+q2I2hQuKNWq3GGWIglR0Ev61UftRxKton3i7huNsJQEKegl3bZt\nC7NAVlVVztDKXPERvYJeEqSgl3SbNw/2EdaIPSzpYtohDvo3CdM4iCSgoKA3s5FmtszM6s1sYjPP\njzKzRWa2wMzmm9nFxS9VOqQXXgjtuecmW0d7HQkMGAC7aRwiKlJmrQa9mVUBk4BLgEHAWDMblLfb\nH4Gh7j4MuAaYUuQ6paN68cXQVmrQA3zsY6GdMyfZOqTDKuSIfjhQ7+4r3X03MBUYlbuDu//Z/S8z\nN3UDNhS3TOmQ3BvDcf01iZZyUM47L7TPPx/aNM+6KZlUSND3JpwOi62JHmvCzEab2WvAE8BXm3sh\nMxtnZnVmVrd+/fr21CsdyZo18M470BXolXQxB0FH9JKwop2MdfdH3X0gcBlwv5nt99ruPsXda9y9\nprq6ulhvLVkVd9t8hMoeNjBoUFjjdtUqePvtpKuRDqiQX5+1wAk59/tEjzXL3WcDnYDjDq406fDi\nE7GVOH4+V1VV48VeOqqXBBQS9HOBAWbWz8w6A2OAabk7mNlHzMyi7bMAc3f1zcjBifu0K/GK2Hyn\nRG38bxIpo06t7eDuDWY2AZgJVAH3uPsSMxsfPT8Z+DvgKjPbA2wj/DEQab8dO8Ksj2ZwSgZWaBoQ\ntXPmwNmJViIdUKtBD+Du04HpeY9Nztm+Hbi9uKVJh/bii+ECo76Ek7GVrj/hj9a8eaDrpqTMKvkU\nl2TZ7NmhPTXZMoqi1sKg40GDYPduTYcgZaegl3SKg35gsmUU1YgRoX0t2TKk41HQS/rs2dM4OiUL\nR/Sxiy4K7auJViEdkIJe0mf+fNi+HU49FY5OupgiuvDC0C4H7s+5OlZXykqJKeglfZ59NrRxV0dW\n9OwJp50Gu1A/vZSVgl7SJ+6fv+CCZOsohbj7ZmmiVUgHo6CXdNmzB55+OmxvvirRUkpCQS8JUNBL\nusydGxbTPp5sTqIR99O/jhYikbJR0Eu6PPVUaM9ItoyS6dkTPkzop3/ppaSrkQ5CQS/p8uSToR2c\nbBklU2twerQ9a1aipUjHoaCX9PjggzBjZVVVWMssa+JhlPGnlZkzEytFOhYFvaTH7NnQ0ADDh2dj\nfpuWDCLMMvXSS7BBi7FJ6SnoJT3i/vlPfCLZOkrtMMIVv+6N/2aRElLQS3rMmBHarAc9qPtGykpB\nL+mwfDksWwbduzeusZplQ6J25kzIwHT7km4KekmH//zP0F5yCXQqaJmEynYi0J2whuzqpIuRrCso\n6M1spJktM7N6M5vYzPOfN7NFZvaKmT1vZkOLX6pk2u9/H9rLLku2jnIxIP4tmZ9kIdIRtBr0ZlYF\nTAIuIYwXGGtm+YPf3gAudPczgNuAKcUuVDJsy5Yw4qaqCj74XMeZzTFeUnAeHeffLIko5Ih+OFDv\n7ivdfTcwFRiVu4O7P+/um6K7LwB9ilumZNrMmWFY5V/9FRyRdDFldAbQhTCTpUZZSgkVEvS9adqL\nuCZ6rCV/D8xo7gkzG2dmdWZWt379+sKrlGzraN02sc40jr5R942UUFFPxprZxwlBf0tzz7v7FHev\ncfea6urqYr61VKpduzpu0ENj901dolVIxhUS9GuBE3Lu94kea8LMhgB3A6PcXR9EpTBPPhn66E8E\n5mVpgdgCnUk4MbsU2Lw54WIkqwoJ+rnAADPrZ2adgTHAtNwdzOxE4BHgi+7+evHLlMx66KHQnpNs\nGYk5krAA+l7g8ccTLkayqtWgd/cGYAIwk3Dc8ZC7LzGz8WY2PtrtO4TZw+80swVmpg+i0rqdOxvD\nraMGPcB5UfvAA4mWIdlV0JUp7j4dmJ732OSc7euB64tbmmTerFmwdSucRFhopKMaDtxHmPdm3bow\nZ71IEenKWEnOgw+GtiMfzUPovhkC7N0LDz+sMfVSdAp6ScbdBr+tDdvnJltKKsTT+9TWJlqGZJOC\nXpLxIrAbGDEC1FMBZxEunpozB95LuhjJGgW9JOOZqL3uukTLSI3DgJpo++kE65BMUtBL+b32Giwn\nhNuVVyZdTXp8PGqfIUwJIVIkCnopj9wTjPfeG9pzgW7dkqgmnQYSRh9tpnHaZpEiUNBLee3cCffc\nE7ZHJFtK6hjwP6LtKZoAVopHQS/l9eCDsH59GDt/StLFpNAFhKtbZsyAVauSrkYyQkEv5eMOP/1p\n2P4k4QhWmjqScF2BO/zsZ0lXIxmhoJfy+edD4OWXobq68bJ/2d8lUTtliiY6k6JQ0Ev5PBG148eH\nudilef2Aj38c/vxnuOuups/pqllpBwW9lMdqwjyoXbrADTc0Pq7gat43vxnaf7sZdu9OthapeAp6\nKY/HovZLX4L/+nCipVSEkSNh8GDYBNx3X9LVSIVT0EvpLV0apjzoBNzS7OJjks8Mvv3tsD1xXBiW\nKtJOCnopvdtuAwcuBPpo3fiCNYwNa7ttROPq5aAUFPRmNtLMlplZvZlNbOb5gWY2x8x2mdk/Fb9M\nqVgvvRQW1OgEdMAlYdut1sJvZzxDxHe/C9u2JVmRVLBWg97MqoBJhEFfg4CxZjYob7eNwFeBfy96\nhVK53OGL0WTzI4FqdPK1rc4G+gPvvQe33550NVKhCjmiHw7Uu/tKd98NTAVG5e7g7u+5+1xgTwlq\nlEr18MPwOmHc/KhW95bmGPD5aPsHP4B1SRYjlaqQoO9NGBwXWxM9JtKyrVvhn6JevNtug67JllPR\nTgWuugp27YJfJ12MVKKynow1s3FmVmdmdevXry/nW0u53XorrF4dLv7pOr7V3aUVt98ORx4J84FH\nH026GqkwhQT9WsK5/1if6LE2c/cp7l7j7jXV1dXteQmpBM8+C3feCZ06wZeAqqQLyoBeveB73wvb\n11wRJoYTKVAhQT8XGGBm/cysMzAGmFbasqQi1Vrosrn22nB/4sQwS6UUx403huEQWwnTSLjr5LYU\npNWgd/cGYAIwE1gKPOTuS8xsvJmNBzCzXma2BvhH4NtmtsbMjipl4ZJCDnz5y7BiBQwd2njBjxy8\nWoNDDoFxhJW5HnkE7r476aqkQnQqZCd3nw5Mz3tscs72u4QuHenI/guYOjWsGvXQQ/C7w5KuKHuq\ngWuAycCECaC/pVIAXRkrxTF7Ntwbbf/Hf8ApWlWkZC4APkGY7OynqL9eWqWgl/aL+4eXL4fRo2Ev\ncNNN8PnPH/DLpAi+AJxzDmwALr00TGks0gIFvRSuuRN/a9aEmRY3boQzgR/9SCcIy+FQ4LHHQlfO\n3Lnw2c/CHl2vKM1T0Ev7bSIskLFyJdTUhFP2D0anfRT2pderF9xCWH5wxgz4zGfCRVUieRT00j71\n9XBb1PYFZs0Ko0GkdJr743k8IeyPOQYefxwuvxy2by93ZZJyCno5sObC5YUX4LzzwrwrfYGJwIxj\ny1uXNOoHPP10mFNo1qzwKevtt5OuSlJEQS+Fc+CXvwxB8v77MAT434SuAym/3D/CQ4aEkU8nnhim\nhq6pgRdfbHl/6VAU9FKYDz6AXwDXXx9WOxo3Dr6BumvSotZg4ECoq4MRI+Cdd+CCC+D734e9e5Ou\nThKmoJfW/eEPYf3S54AuwHjCWPmCLreTsqm10H3z5JPwN4RROLfeCoM7heUcW/taySwFvbTs9dfh\nJ8Bll4WZKE8mnIC9AAVDmnXuDFcTRuJ0B5YRunZ+DWzZkmxtkggFvezvrbfghhtg0KAwpV3XrnDH\nHfAvaCWCtMv9AzxyJNwOXEzovpkB9O8fZsHcurX47yeppaCXRi+9BGPHwsknw+TJYXbEiwhH9j3/\nUdMNV4o4fGsNjgCuA+bNg4HAhg3wrW/BSSfB5QY/UVB3BAr6ju7dd8PR+plnhkvqp04F3xsCf/Hi\nMJ98bx3GV7ylZ4UJ0J56Ci68EDZvht8T5psdORLmoGkUMkxB39G4hwD/wQ/CL3zv3vCNb8CCBdAN\nuPnm0C9fWwunnRa+Rh/Ps8GAiy8OY+6few7OJ5xQnzkTfk44kTt6NNx/f+M4fH3vM8HcPZE3rqmp\n8bq6ukTeu0P5tcEZC8Iv9nPPhdWfVucsAVwFfGoUfPGL8Ocr4epoMYvPaVGLTPpc9Psef28/AHb9\nFCbdFBZyz3XaaXDCUrhuKnz0o9CvH1jez0T8Op9LJkc6IjOb5+41bfkaDZArlzg8S/Va7mG62qVL\n4ZVXGm8vAzuHNd33KGD01WHWwy2fha6Pw989BrUo3DuC3O/xkcCXvwo9boKNgP8MnngCnnkm/Cwt\nBWaNCfsed1y4EGvoUNj8A7h2Dmyn6cLvxfw5l6Ip6IjezEYSZr6uAu529+/nPW/R839L+NZf4+7z\nD/SaHe6I/mB+AdzDsLh16+DXA2ELcPKP4Y03mt62bWv+6086CXqvglOAry2AV4bt32mnI/iOobXv\nc/z8lbvCyfmfXAArgJWEJQyb0x0YeC74C9ADOA4Y/Sj06QPPfRS+tC2M3JKiKMkRvZlVAZOAvwbW\nAHPNbJq7v5qz2yXAgOh2DuEaynPaUkgmNTTAjh3hth547bXG+zt2hKtNt2wJJ8a2bNl/e9MmeO+9\nEPC7d+e9+Nf3f7/u3WHAADjjjHB77+thWfcb3mz85R46FJaU9p8tKVboH/POneH88+Gt6L4D578Z\nrrx94Ep4G9gxDF5dAJsJ8x/lun904/bXuoVVx6qrG2/HHgtHHdXy7Ygj4PDDG2+HHQbTjoOr9+3f\nfSStKqTrZjhQ7+4rAcxsKjAKyA36UcD9Hj4evGBm3c3seHd/p8VX3bwZHn0U9u0LR6z79jXdLvSx\nYnxNQ0O4ijBu4+3l90OfzzR9rqV2w2JoAA7pCVvWQUOn8FwTp7Xle9PUEUdAz55QtQKOBoZ/BTZN\nCvORfwi4cVMIesjpY/964/1YS7/oOpqXXPk/DwY81zf8XP1lJuQFsI/Q5bMBeD9q4+1NhE8B2zqH\nT5vbtsGbbx5cXdceAl26NP0j0KULHHpo461Tp/2333kc+o9t/rmqqrAeb0u31p6f91U4586Wvy7+\nw2TW+i1/v9mfhoum7b9PGxUS9L2BnLN3rGH/o/Xm9ukNtBz0K1bAFVcUVmWiHm7j/uuitiH8cnQG\njjgOfAMcNwB2LQ+LRpxwEWx9Gg4njHY5nNDX2RX4mwdh/v8Mjx0NjIs++jb55ZvU9G27d1dYy8Fr\n68/QIYTumh7AqS3sM3ZnuEDrl93Dyd+thA7eHdGt3y0w7/awfexlUP97OPwseHc+7CHcdtO4vWtX\nuG3e3LZaX3qgbfu3xX03lu6177j8oF+i1T56M7sSGOnu10f3vwic4+4Tcvb5A/B9d/9TdP+PwC3u\nXpf3WuMI69gDnA4sPuh/QXH1IByLpE0a61JNhVFNhUtjXWms6VR3b9OcsYUc0a8l9PTG+kSPtXUf\n3H0KMAXAzOraekKh1NJYE6SzLtVUGNVUuDTWldaa2vo1hVwwNRcYYGb9zKwzMAaYlrfPNOAqC84F\nthywf15ERMqm1SN6d28wswnATMLwynvcfYmZjY+enwxMJwytrCf0vl1bupJFRKQtCrpgyt2nE8I8\n97HJOdsOfKWN7z2ljfuXQxprgnTWpZoKo5oKl8a6MlFTYlMgiIhIeWhSMxGRjEsk6KMLqn5rZq+Z\n2VIzOy+JOnLqOdXMFuTctprZ15KsKarrVjN71cwWm9kDZpb4Cq1mdlNUz5Ik/4/M7B4ze8/MFuc8\ndqyZPWlmy6P2mBTU9Jno/2qfmZV99EYLNf0w+t1bZGaPmln3FNR0W1TPQjP7/2Z2YjlraqmunOe+\nYWZuZj2ccYrlAAADmklEQVSSrsnM/sXM1ubk1d+29jpJHdH/FHjC3QcCQwlTJyXG3Ze5+zB3Hwac\nTTih/GiSNZlZX8I1B2e7++mEE+FjEq7pdMIM9cMJ37dPmdlHEirnXmBk3mMTgT+6+wDgj9H9pGta\nDFwBzC5zLbF72b+mJ4HT3X0IYc7KW1NQ0w/dfYi7DwUeA/65zDVB83VhZicQVuF9K/+5MriXZmoC\nfhxnVnQO9YDKHvRmdjQwAvglgLvvdvc2XuJWUhcDK9x9VcJ1bCVcB3i4mXUiXDP7drIlcRrwortv\nd/cG4BlCiJWdu88mXHyfaxRwX7R9H/DppGty96XuvqycdeS9f3M1zYq+fwAvEK57Sbqm3CnTuhEm\nUiirFn6mAH4M3EyY8aesDlBTmyRxRN+PMMXXr8zsZTO728y6JVBHS8YAJbxWujDuvhH4d8JRxDuE\naxNmJVsVi4ELzOw4M+tKGFJ7QitfU049c67feBfomWQxFeI6wmqyiTOz75rZasLw7H9Luh4AMxsF\nrHX3hUnXkucfoq6uewrpokwi6DsBZwG/cPczgW2U/yN2s6ILwi6n7RPclKKW/oQpKvsBHwa6mdkX\nkqzJ3ZcSlpueBTwBLAD2JllTS6IhvxpSdgBm9i3CVHy/SboWAHf/lrufAPyKcBSdqOhg5n8B30m6\nljy/AE4GhhEOAn/U2hckEfRrgDXu/mJ0/7eE4E+DS4D57r6u1T1LrwZ43t3Xu/se4BHgYwnXhLv/\n0t3PdvcRhPkJ89clStI6MzseIGrfS7ie1DKza4BPAZ/39I2x/g3w0aSLAPoTDrQWmtmbhC6u+WbW\nK8mi3H2du+91933AXYRzZgdU9qB393eB1WYWz3V3MU2nPE7SWFLQbRNZBpxrZl2jhV0uJuGT1gBm\n9qGoPZHQP1+bbEVNTAOujravBh5PsJbUsrCQ0M3A5e6+Pel6AMxsQM7dUYRPi4ly91fc/UPu3tfd\n+xIOUs+KMiwx8cFMZDSFTA7p7mW/ET5y1AGLCGfYj0mijrya4hNARyddS05NtxD+CC4G/h/QJQU1\nPRvVtBC4OME6HiB8bN1D+AX8e8LaRn8ElgNPAcemoKbR0fYuwhzWM1NQUz1hWvEF0W1yCmr6XfRz\nvpAw4q1XGn6m8p5/E+iRdE1RFrwS5ec04PjWXkdXxoqIZJyujBURyTgFvYhIxinoRUQyTkEvIpJx\nCnoRkYxT0IuIZJyCXkQk4xT0IiIZ99/lzkHXGjQFpQAAAABJRU5ErkJggg==\n", + "image/png": "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\n", "text/plain": [ - "" + "
" ] }, "metadata": {}, @@ -158,18 +149,14 @@ "mus=[np.sqrt(y/5)*std_norm.rvs()+xbar for y in samps]\n", "fig,ax=plt.subplots(1,1)\n", "x=np.linspace(6,15,1000)\n", - "#y=t.pdf(x,4,loc=xbar,scale=np.sqrt(v/5))\n", "plt.xlim(6,15)\n", "junk=ax.plot(x,t.pdf(x,4,loc=xbar,scale=np.sqrt(v/5)),lw=2,color='red')\n", - "junk=ax.hist([x for x in mus if x>6 or x<15],bins=500,normed=True,color='orange')\n" + "junk=ax.hist([x for x in mus if x>6 or x<15],bins=500,density=True,color='orange')\n" ] }, { "cell_type": "markdown", - "metadata": { - "deletable": true, - "editable": true - }, + "metadata": {}, "source": [ "If we treat the measurements as rounded, then a result of \"10\" means that the true result lies between 9.5 and 10.5.\n", "To compute $p(\\mu,\\sigma^2|y)$ we need the likelihood $p(y|\\mu,\\sigma^2)$ and this comes from normal hypothesis.\n", @@ -192,37 +179,48 @@ }, { "cell_type": "code", - "execution_count": 20, - "metadata": { - "collapsed": true, - "deletable": true, - "editable": true - }, + "execution_count": 15, + "metadata": {}, "outputs": [], "source": [ + "# modified to follow Gelman\n", + "def L1(mu,sigma,dx):\n", + " l=0\n", + " for x in dx:\n", + " try:\n", + " l=l+np.log(norm.cdf(x+.5,loc=mu,scale=sigma)-norm.cdf(x-.5,loc=mu,scale=sigma))\n", + " except:\n", + " l=l-10000\n", + " return l\n", "\n", - "def pfun(mu,sigma,n):\n", - " return norm.cdf(n+.5,loc=mu,scale=sigma)-norm.cdf(n-.5,loc=mu,scale=sigma)\n", - "def Lm(mu,sigma):\n", - " return pfun(mu,sigma,10)**2*pfun(mu,sigma,11)*pfun(mu,sigma,9)*pfun(mu,sigma,12)\n", - "def L(mu,sigma):\n", - " return -log(Lm(mu,sigma))+2*log(sigma)\n" + "def L2(mu,sigma,dx):\n", + " l=0\n", + " for x in dx:\n", + " try:\n", + " l=l+np.log(norm.pdf(x,loc=mu,scale=sigma))\n", + " except:\n", + " l=l-10000\n", + " return l" ] }, { "cell_type": "code", - "execution_count": 23, - "metadata": { - "collapsed": false, - "deletable": true, - "editable": true - }, + "execution_count": 16, + "metadata": {}, "outputs": [ + { + "name": "stderr", + "output_type": "stream", + "text": [ + "/Users/jteitelbaum/anaconda3/lib/python3.6/site-packages/ipykernel_launcher.py:6: RuntimeWarning: divide by zero encountered in log\n", + " \n" + ] + }, { "data": { - "image/png": "iVBORw0KGgoAAAANSUhEUgAAAX8AAAD8CAYAAACfF6SlAAAABHNCSVQICAgIfAhkiAAAAAlwSFlz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\nD/L/AcAlRd+XBVkKAAAAAElFTkSuQmCC\n", + "image/png": "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\n", "text/plain": [ - "" + "
" ] }, "metadata": {}, @@ -230,28 +228,36 @@ } ], "source": [ - "u=np.linspace(8,12,200)\n", - "v=np.linspace(.2,4,200)\n", - "X,Y=np.meshgrid(u,v)\n", - "Z1=np.vectorize(L)(X,np.sqrt(Y))\n", - "CS=plt.contour(X,np.sqrt(Y),Z1,levels=np.arange(.5,15,.25))\n", - "\n" + "u=np.linspace(3,18,200)\n", + "v=np.linspace(-2,4,200)\n", + "X,Y=np.meshgrid(u,np.exp(v))\n", + "#print(np.exp(Y))\n", + "Z1=L1(X,Y,[10,10,11,9,12])\n", + "lev=[.0001,.001,.01]+[x for x in np.arange(.05,.95,.05)]\n", + "CS=plt.contour(X,np.log(Y),np.exp(Z1-np.max(Z1)),levels=lev)\n", + "junk=plt.clabel(CS,[.0001,.001,.01],fmt=\"%1.4f\")\n" ] }, { "cell_type": "code", - "execution_count": 22, + "execution_count": 17, "metadata": { - "collapsed": false, - "deletable": true, - "editable": true + "scrolled": true }, "outputs": [ + { + "name": "stderr", + "output_type": "stream", + "text": [ + "/Users/jteitelbaum/anaconda3/lib/python3.6/site-packages/ipykernel_launcher.py:15: RuntimeWarning: divide by zero encountered in log\n", + " from ipykernel import kernelapp as app\n" + ] + }, { "data": { - "image/png": "iVBORw0KGgoAAAANSUhEUgAAAX8AAAD8CAYAAACfF6SlAAAABHNCSVQICAgIfAhkiAAAAAlwSFlz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\n2wAAAABJRU5ErkJggg==\n", + "image/png": "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\n", "text/plain": [ - "" + "
" ] }, "metadata": {}, @@ -259,54 +265,316 @@ } ], "source": [ - "u=np.linspace(8,12,200)\n", - "v=np.linspace(.2,4,200)\n", - "X,Y=np.meshgrid(u,v)\n", - "Z2=(4*(1.3)+5*(10.4-X)**2)/2/Y+7.0/2*np.log(Y)\n", - "CS=plt.contour(X,np.sqrt(Y),Z2,levels=np.arange(.5,15,.25))\n", + "u=np.linspace(3,18,200)\n", + "v=np.linspace(-2,4,200)\n", + "X,Y=np.meshgrid(u,np.exp(v))\n", + "Z2=L2(X,Y,[10,10,11,9,12])\n", + "CS=plt.contour(X,np.log(Y),np.exp(Z2-np.max(Z2)),levels=lev)\n", + "junk=plt.clabel(CS,[.0001,.001,.01],fmt=\"%1.4f\")\n" + ] + }, + { + "cell_type": "markdown", + "metadata": {}, + "source": [ + "What is the posterior mode for sigma? It follows Inv-chi^2(n-1,s^2) which has mode (n-1)/(n+1)s^2, or in our case (2/3)s^2 or about .866." + ] + }, + { + "cell_type": "code", + "execution_count": 225, + "metadata": {}, + "outputs": [ + { + "data": { + "text/plain": [ + "array([[-2. , -2. , -2. , ..., -2. ,\n", + " -2. , -2. ],\n", + " [-1.9798995 , -1.9798995 , -1.9798995 , ..., -1.9798995 ,\n", + " -1.9798995 , -1.9798995 ],\n", + " [-1.95979899, -1.95979899, -1.95979899, ..., -1.95979899,\n", + " -1.95979899, -1.95979899],\n", + " ...,\n", + " [ 1.95979899, 1.95979899, 1.95979899, ..., 1.95979899,\n", + " 1.95979899, 1.95979899],\n", + " [ 1.9798995 , 1.9798995 , 1.9798995 , ..., 1.9798995 ,\n", + " 1.9798995 , 1.9798995 ],\n", + " [ 2. , 2. , 2. , ..., 2. ,\n", + " 2. , 2. ]])" + ] + }, + "execution_count": 225, + "metadata": {}, + "output_type": "execute_result" + } + ], + "source": [ + "Y\n", "\n" ] }, { "cell_type": "code", - "execution_count": null, - "metadata": { - "collapsed": true, - "deletable": true, - "editable": true - }, - "outputs": [], - "source": [] + "execution_count": 231, + "metadata": {}, + "outputs": [ + { + "name": "stdout", + "output_type": "stream", + "text": [ + "[0.82616995 1.33837647 1.80934486 2.49570902 5.68990452]\n" + ] + } + ], + "source": [ + "Psigma=list(map(sum,np.exp(Z2-np.max(Z2))))\n", + "p=Psigma/sum(Psigma)\n", + "s=np.random.choice(np.exp(np.linspace(-2,2,200)),p=p,size=1000)\n", + "print(np.percentile(s,q=[2.5,25,50,75,97.5]))" + ] }, { - "cell_type": "markdown", - "metadata": { - "deletable": true, - "editable": true - }, + "cell_type": "code", + "execution_count": 178, + "metadata": {}, + "outputs": [], "source": [ - "What is the posterior mode for sigma? It follows Inv-chi^2(n-1,s^2) which has mode (n-1)/(n+1)s^2, or in our case (2/3)s^2 or about .866." + "p=[1/3,1/3].append([1/200.0]*198)" ] }, { "cell_type": "code", - "execution_count": null, - "metadata": { - "collapsed": true, - "deletable": true, - "editable": true - }, + "execution_count": 180, + "metadata": {}, "outputs": [], - "source": [] + "source": [ + "p=[1/3,1/3]\n", + "p.append([1/200.0]*198)\n" + ] + }, + { + "cell_type": "code", + "execution_count": 181, + "metadata": {}, + "outputs": [ + { + "data": { + "text/plain": [ + "[0.3333333333333333,\n", + " 0.3333333333333333,\n", + " [0.005,\n", + " 0.005,\n", + " 0.005,\n", + " 0.005,\n", + " 0.005,\n", + " 0.005,\n", + " 0.005,\n", + " 0.005,\n", + " 0.005,\n", + " 0.005,\n", + " 0.005,\n", + " 0.005,\n", + " 0.005,\n", + " 0.005,\n", + " 0.005,\n", + " 0.005,\n", + " 0.005,\n", + " 0.005,\n", + " 0.005,\n", + " 0.005,\n", + " 0.005,\n", + " 0.005,\n", + " 0.005,\n", + " 0.005,\n", + " 0.005,\n", + " 0.005,\n", + " 0.005,\n", + " 0.005,\n", + " 0.005,\n", + " 0.005,\n", + " 0.005,\n", + " 0.005,\n", + " 0.005,\n", + " 0.005,\n", + " 0.005,\n", + " 0.005,\n", + " 0.005,\n", + " 0.005,\n", + " 0.005,\n", + " 0.005,\n", + " 0.005,\n", + " 0.005,\n", + " 0.005,\n", + " 0.005,\n", + " 0.005,\n", + " 0.005,\n", + " 0.005,\n", + " 0.005,\n", + " 0.005,\n", + " 0.005,\n", + " 0.005,\n", + " 0.005,\n", + " 0.005,\n", + " 0.005,\n", + " 0.005,\n", + " 0.005,\n", + " 0.005,\n", + " 0.005,\n", + " 0.005,\n", + " 0.005,\n", + " 0.005,\n", + " 0.005,\n", + " 0.005,\n", + " 0.005,\n", + " 0.005,\n", + " 0.005,\n", + " 0.005,\n", + " 0.005,\n", + " 0.005,\n", + " 0.005,\n", + " 0.005,\n", + " 0.005,\n", + " 0.005,\n", + " 0.005,\n", + " 0.005,\n", + " 0.005,\n", + " 0.005,\n", + " 0.005,\n", + " 0.005,\n", + " 0.005,\n", + " 0.005,\n", + " 0.005,\n", + " 0.005,\n", + " 0.005,\n", + " 0.005,\n", + " 0.005,\n", + " 0.005,\n", + " 0.005,\n", + " 0.005,\n", + " 0.005,\n", + " 0.005,\n", + " 0.005,\n", + " 0.005,\n", + " 0.005,\n", + " 0.005,\n", + " 0.005,\n", + " 0.005,\n", + " 0.005,\n", + " 0.005,\n", + " 0.005,\n", + " 0.005,\n", + " 0.005,\n", + " 0.005,\n", + " 0.005,\n", + " 0.005,\n", + " 0.005,\n", + " 0.005,\n", + " 0.005,\n", + " 0.005,\n", + " 0.005,\n", + " 0.005,\n", + " 0.005,\n", + " 0.005,\n", + " 0.005,\n", + " 0.005,\n", + " 0.005,\n", + " 0.005,\n", + " 0.005,\n", + " 0.005,\n", + " 0.005,\n", + " 0.005,\n", + " 0.005,\n", + " 0.005,\n", + " 0.005,\n", + " 0.005,\n", + " 0.005,\n", + " 0.005,\n", + " 0.005,\n", + " 0.005,\n", + " 0.005,\n", + " 0.005,\n", + " 0.005,\n", + " 0.005,\n", + " 0.005,\n", + " 0.005,\n", + " 0.005,\n", + " 0.005,\n", + " 0.005,\n", + " 0.005,\n", + " 0.005,\n", + " 0.005,\n", + " 0.005,\n", + " 0.005,\n", + " 0.005,\n", + " 0.005,\n", + " 0.005,\n", + " 0.005,\n", + " 0.005,\n", + " 0.005,\n", + " 0.005,\n", + " 0.005,\n", + " 0.005,\n", + " 0.005,\n", + " 0.005,\n", + " 0.005,\n", + " 0.005,\n", + " 0.005,\n", + " 0.005,\n", + " 0.005,\n", + " 0.005,\n", + " 0.005,\n", + " 0.005,\n", + " 0.005,\n", + " 0.005,\n", + " 0.005,\n", + " 0.005,\n", + " 0.005,\n", + " 0.005,\n", + " 0.005,\n", + " 0.005,\n", + " 0.005,\n", + " 0.005,\n", + " 0.005,\n", + " 0.005,\n", + " 0.005,\n", + " 0.005,\n", + " 0.005,\n", + " 0.005,\n", + " 0.005,\n", + " 0.005,\n", + " 0.005,\n", + " 0.005,\n", + " 0.005,\n", + " 0.005,\n", + " 0.005,\n", + " 0.005,\n", + " 0.005,\n", + " 0.005,\n", + " 0.005,\n", + " 0.005,\n", + " 0.005,\n", + " 0.005,\n", + " 0.005,\n", + " 0.005,\n", + " 0.005,\n", + " 0.005,\n", + " 0.005,\n", + " 0.005]]" + ] + }, + "execution_count": 181, + "metadata": {}, + "output_type": "execute_result" + } + ], + "source": [ + "p" + ] }, { "cell_type": "code", "execution_count": null, - "metadata": { - "collapsed": true, - "deletable": true, - "editable": true - }, + "metadata": {}, "outputs": [], "source": [] } @@ -327,7 +595,7 @@ "name": "python", "nbconvert_exporter": "python", "pygments_lexer": "ipython3", - "version": "3.5.2" + "version": "3.6.4" } }, "nbformat": 4, diff --git a/gelman.r b/gelman.r new file mode 100644 index 0000000..f39d03f --- /dev/null +++ b/gelman.r @@ -0,0 +1,36 @@ +post.a <- function(mu,sd,y){ +ldens <- 0 +for (i in 1:length(y)) ldens <- ldens + log(dnorm(y[i],mu,sd)) +ldens} + +post.b <- function(mu,sd,y){ + ldens <- 0 + for (i in 1:length(y)) ldens <- ldens + + log(pnorm(y[i]+0.5,mu,sd) - pnorm(y[i]-0.5,mu,sd)) +ldens} + summ <- function(x){c(mean(x),sqrt(var(x)), quantile(x, c(.025,.25,.5,.75,.975)))} + nsim <- 2000 + y <- c(10,10,12,11,9) + n <- length(y) + ybar <- mean(y) + s2 <- sum((y-mean(y))^2)/(n-1) + mugrid <- seq(3,18,length=200) + logsdgrid <- seq(-2,4,length=200) + contours <- c(.0001,.001,.01,seq(.05,.95,.05)) + logdens <- outer (mugrid, exp(logsdgrid), post.a, y) + dens <- exp(logdens - max(logdens)) + contour (mugrid, logsdgrid, dens, levels=contours, xlab="mu", ylab="log sigma", labex=0, cex=2) + mtext ("Posterior density, ignoring rounding", 3) + sd <- sqrt((n-1)*s2/rchisq(nsim,4)) + mu <- rnorm(nsim,ybar,sd/sqrt(n)) + print (rbind (summ(mu),summ(sd))) + logdens <- outer (mugrid, exp(logsdgrid), post.b, y) + dens <- exp(logdens - max(logdens)) + contour (mugrid, logsdgrid, dens, levels=contours,xlab="mu", ylab="log sigma", labex=0, cex=2) + mtext ("Posterior density, accounting for rounding", cex=2, 3) + dens.mu <- apply(dens,1,sum) + muindex <- sample (1:length(mugrid), nsim, replace=T, prob=dens.mu) + mu <- mugrid[muindex] + sd <- rep (NA,nsim) + for (i in (1:nsim)) sd[i] <- exp (sample(logsdgrid, 1, prob=dens[muindex[i],])) + kprint (rbind (summ(mu),summ(sd))) \ No newline at end of file